Forming an orthonormal vector when you already have two perpendicular vectors

The name of the pictureThe name of the pictureThe name of the pictureClash Royale CLAN TAG#URR8PPP











up vote
4
down vote

favorite












So I understand the requirements for an orthonormal basis and everything around it. However, there's one thing I am missing:



Suppose you have two vectors which are orthonormal $u_1$ and $u_2$. According to the answerbook the multiplication of vector $u_1$ and $u_2$ results in another orthonormal vector $u_3$.



Is this an actual standard theory? Does the multiplication of two orthonormal vectors results in another orthonormal vector?



I have included a picture just to make it more clear.



Thank you in advance :)



enter image description here










share|cite|improve this question























  • Yes, for two vectors in $mathbb R^3$ which are orthonormal, the cross product always gives you a third vector making the three vectors orthonormal.
    – Kavi Rama Murthy
    3 hours ago















up vote
4
down vote

favorite












So I understand the requirements for an orthonormal basis and everything around it. However, there's one thing I am missing:



Suppose you have two vectors which are orthonormal $u_1$ and $u_2$. According to the answerbook the multiplication of vector $u_1$ and $u_2$ results in another orthonormal vector $u_3$.



Is this an actual standard theory? Does the multiplication of two orthonormal vectors results in another orthonormal vector?



I have included a picture just to make it more clear.



Thank you in advance :)



enter image description here










share|cite|improve this question























  • Yes, for two vectors in $mathbb R^3$ which are orthonormal, the cross product always gives you a third vector making the three vectors orthonormal.
    – Kavi Rama Murthy
    3 hours ago













up vote
4
down vote

favorite









up vote
4
down vote

favorite











So I understand the requirements for an orthonormal basis and everything around it. However, there's one thing I am missing:



Suppose you have two vectors which are orthonormal $u_1$ and $u_2$. According to the answerbook the multiplication of vector $u_1$ and $u_2$ results in another orthonormal vector $u_3$.



Is this an actual standard theory? Does the multiplication of two orthonormal vectors results in another orthonormal vector?



I have included a picture just to make it more clear.



Thank you in advance :)



enter image description here










share|cite|improve this question















So I understand the requirements for an orthonormal basis and everything around it. However, there's one thing I am missing:



Suppose you have two vectors which are orthonormal $u_1$ and $u_2$. According to the answerbook the multiplication of vector $u_1$ and $u_2$ results in another orthonormal vector $u_3$.



Is this an actual standard theory? Does the multiplication of two orthonormal vectors results in another orthonormal vector?



I have included a picture just to make it more clear.



Thank you in advance :)



enter image description here







linear-algebra orthonormal cross-product






share|cite|improve this question















share|cite|improve this question













share|cite|improve this question




share|cite|improve this question








edited 2 hours ago









mechanodroid

23.7k52144




23.7k52144










asked 3 hours ago









MathNoob123

311




311











  • Yes, for two vectors in $mathbb R^3$ which are orthonormal, the cross product always gives you a third vector making the three vectors orthonormal.
    – Kavi Rama Murthy
    3 hours ago

















  • Yes, for two vectors in $mathbb R^3$ which are orthonormal, the cross product always gives you a third vector making the three vectors orthonormal.
    – Kavi Rama Murthy
    3 hours ago
















Yes, for two vectors in $mathbb R^3$ which are orthonormal, the cross product always gives you a third vector making the three vectors orthonormal.
– Kavi Rama Murthy
3 hours ago





Yes, for two vectors in $mathbb R^3$ which are orthonormal, the cross product always gives you a third vector making the three vectors orthonormal.
– Kavi Rama Murthy
3 hours ago











3 Answers
3






active

oldest

votes

















up vote
2
down vote













Here we are using the property of cross product which is defined only for $vin mathbbR^3$.



enter image description here



The method is therefore not useful in general but it is very effective in that case to find an orthonormal basis.






share|cite|improve this answer



























    up vote
    2
    down vote













    The cross-product $u_1times u_2$ of any two vectors $u_1$ and $u_2$ is always orthogonal to both of them. Furthermore,$$lVert u_1times u_2rVert=lVert u_1rVert.lVert u_2rVert.sintheta,$$where $theta$ is the angle between them. Therefore, if $u_1$ and $u_2$ are orthogonal and both of them have norm $1$, $u_1times u_2$ will also have norm $1$ (and it will be orthogonal to the other two).






    share|cite|improve this answer



























      up vote
      1
      down vote













      Yes, you can check it by direct calculation.



      Assume that $u = beginbmatrix u_1 \ u_2 \ u_3 endbmatrix$ and $v = beginbmatrix v_1 \ v_2 \ v_3 endbmatrix$ are orthonormal vectors. Their cross product is defined as
      $$u times v = beginbmatrix u_2v_3 - u_3v_2 \ u_3v_1-u_1v_3 \ u_1v_2 - u_2v_1 endbmatrix$$



      We have



      $$langle u, u times vrangle = u_1u_2v_3 - u_1u_3v_2 + u_2u_3v_1 - u_1u_2v_3 + u_1u_3v_2 - u_2u_3v_1 = 0$$



      $$langle v, u times vrangle = u_2v_1v_3 - u_3v_1v_2 + u_2v_1v_2 - u_1v_2v_3 + u_1v_2v_3 - u_2v_1v_3 = 0$$



      beginalign
      |utimes v|^2 &= (u_2v_3 - u_3v_2)^2 + (u_3v_1-u_1v_3)^2 + (u_1v_2 - u_2v_1)^2 \
      &= u_2^2v_3^2 - 2u_2u_3v_2v_3 + u_3^2v_2^2 + u_3^2v_1^2 - 2u_1u_3v_1v_3 + u_1^2v_3^2 + u_1^2v_2^2 - 2u_1u_2v_1v_2 + u_2^2v_1^2 \
      &= u_1^2(v_2^2 + v_3^2) + u_2^2(v_1^2 + v_3^2) + u_3^2(v_1^2 + v_2^2) - 2u_2u_3v_2v_3 - 2u_1u_3v_1v_3 - 2u_1u_2v_1v_2 \
      &= u_1^2(1-v_1^2) + u_2^2(1 -v_2^2) + u_3^2(1-v_3^2) - 2u_2u_3v_2v_3 - 2u_1u_3v_1v_3 - 2u_1u_2v_1v_2 \
      &= (u_1^2+u_2^2+u_3^2) - (u_1^2v_1^2 + u_2^2v_2^2+u_3^2v_3^2 + 2u_1u_2v_1v_2 + 2u_1u_3v_1v_3 + 2u_2u_3v_2v_3)\
      &= |u|^2 - langle u,vrangle ^2\
      &= 1
      endalign



      so $u, v, utimes v$ is an orthonormal basis for $mathbbR^3$.






      share|cite|improve this answer




















        Your Answer




        StackExchange.ifUsing("editor", function ()
        return StackExchange.using("mathjaxEditing", function ()
        StackExchange.MarkdownEditor.creationCallbacks.add(function (editor, postfix)
        StackExchange.mathjaxEditing.prepareWmdForMathJax(editor, postfix, [["$", "$"], ["\\(","\\)"]]);
        );
        );
        , "mathjax-editing");

        StackExchange.ready(function()
        var channelOptions =
        tags: "".split(" "),
        id: "69"
        ;
        initTagRenderer("".split(" "), "".split(" "), channelOptions);

        StackExchange.using("externalEditor", function()
        // Have to fire editor after snippets, if snippets enabled
        if (StackExchange.settings.snippets.snippetsEnabled)
        StackExchange.using("snippets", function()
        createEditor();
        );

        else
        createEditor();

        );

        function createEditor()
        StackExchange.prepareEditor(
        heartbeatType: 'answer',
        convertImagesToLinks: true,
        noModals: false,
        showLowRepImageUploadWarning: true,
        reputationToPostImages: 10,
        bindNavPrevention: true,
        postfix: "",
        noCode: true, onDemand: true,
        discardSelector: ".discard-answer"
        ,immediatelyShowMarkdownHelp:true
        );



        );













         

        draft saved


        draft discarded


















        StackExchange.ready(
        function ()
        StackExchange.openid.initPostLogin('.new-post-login', 'https%3a%2f%2fmath.stackexchange.com%2fquestions%2f2916600%2fforming-an-orthonormal-vector-when-you-already-have-two-perpendicular-vectors%23new-answer', 'question_page');

        );

        Post as a guest






























        3 Answers
        3






        active

        oldest

        votes








        3 Answers
        3






        active

        oldest

        votes









        active

        oldest

        votes






        active

        oldest

        votes








        up vote
        2
        down vote













        Here we are using the property of cross product which is defined only for $vin mathbbR^3$.



        enter image description here



        The method is therefore not useful in general but it is very effective in that case to find an orthonormal basis.






        share|cite|improve this answer
























          up vote
          2
          down vote













          Here we are using the property of cross product which is defined only for $vin mathbbR^3$.



          enter image description here



          The method is therefore not useful in general but it is very effective in that case to find an orthonormal basis.






          share|cite|improve this answer






















            up vote
            2
            down vote










            up vote
            2
            down vote









            Here we are using the property of cross product which is defined only for $vin mathbbR^3$.



            enter image description here



            The method is therefore not useful in general but it is very effective in that case to find an orthonormal basis.






            share|cite|improve this answer












            Here we are using the property of cross product which is defined only for $vin mathbbR^3$.



            enter image description here



            The method is therefore not useful in general but it is very effective in that case to find an orthonormal basis.







            share|cite|improve this answer












            share|cite|improve this answer



            share|cite|improve this answer










            answered 3 hours ago









            gimusi

            71.8k73888




            71.8k73888




















                up vote
                2
                down vote













                The cross-product $u_1times u_2$ of any two vectors $u_1$ and $u_2$ is always orthogonal to both of them. Furthermore,$$lVert u_1times u_2rVert=lVert u_1rVert.lVert u_2rVert.sintheta,$$where $theta$ is the angle between them. Therefore, if $u_1$ and $u_2$ are orthogonal and both of them have norm $1$, $u_1times u_2$ will also have norm $1$ (and it will be orthogonal to the other two).






                share|cite|improve this answer
























                  up vote
                  2
                  down vote













                  The cross-product $u_1times u_2$ of any two vectors $u_1$ and $u_2$ is always orthogonal to both of them. Furthermore,$$lVert u_1times u_2rVert=lVert u_1rVert.lVert u_2rVert.sintheta,$$where $theta$ is the angle between them. Therefore, if $u_1$ and $u_2$ are orthogonal and both of them have norm $1$, $u_1times u_2$ will also have norm $1$ (and it will be orthogonal to the other two).






                  share|cite|improve this answer






















                    up vote
                    2
                    down vote










                    up vote
                    2
                    down vote









                    The cross-product $u_1times u_2$ of any two vectors $u_1$ and $u_2$ is always orthogonal to both of them. Furthermore,$$lVert u_1times u_2rVert=lVert u_1rVert.lVert u_2rVert.sintheta,$$where $theta$ is the angle between them. Therefore, if $u_1$ and $u_2$ are orthogonal and both of them have norm $1$, $u_1times u_2$ will also have norm $1$ (and it will be orthogonal to the other two).






                    share|cite|improve this answer












                    The cross-product $u_1times u_2$ of any two vectors $u_1$ and $u_2$ is always orthogonal to both of them. Furthermore,$$lVert u_1times u_2rVert=lVert u_1rVert.lVert u_2rVert.sintheta,$$where $theta$ is the angle between them. Therefore, if $u_1$ and $u_2$ are orthogonal and both of them have norm $1$, $u_1times u_2$ will also have norm $1$ (and it will be orthogonal to the other two).







                    share|cite|improve this answer












                    share|cite|improve this answer



                    share|cite|improve this answer










                    answered 3 hours ago









                    José Carlos Santos

                    121k16101185




                    121k16101185




















                        up vote
                        1
                        down vote













                        Yes, you can check it by direct calculation.



                        Assume that $u = beginbmatrix u_1 \ u_2 \ u_3 endbmatrix$ and $v = beginbmatrix v_1 \ v_2 \ v_3 endbmatrix$ are orthonormal vectors. Their cross product is defined as
                        $$u times v = beginbmatrix u_2v_3 - u_3v_2 \ u_3v_1-u_1v_3 \ u_1v_2 - u_2v_1 endbmatrix$$



                        We have



                        $$langle u, u times vrangle = u_1u_2v_3 - u_1u_3v_2 + u_2u_3v_1 - u_1u_2v_3 + u_1u_3v_2 - u_2u_3v_1 = 0$$



                        $$langle v, u times vrangle = u_2v_1v_3 - u_3v_1v_2 + u_2v_1v_2 - u_1v_2v_3 + u_1v_2v_3 - u_2v_1v_3 = 0$$



                        beginalign
                        |utimes v|^2 &= (u_2v_3 - u_3v_2)^2 + (u_3v_1-u_1v_3)^2 + (u_1v_2 - u_2v_1)^2 \
                        &= u_2^2v_3^2 - 2u_2u_3v_2v_3 + u_3^2v_2^2 + u_3^2v_1^2 - 2u_1u_3v_1v_3 + u_1^2v_3^2 + u_1^2v_2^2 - 2u_1u_2v_1v_2 + u_2^2v_1^2 \
                        &= u_1^2(v_2^2 + v_3^2) + u_2^2(v_1^2 + v_3^2) + u_3^2(v_1^2 + v_2^2) - 2u_2u_3v_2v_3 - 2u_1u_3v_1v_3 - 2u_1u_2v_1v_2 \
                        &= u_1^2(1-v_1^2) + u_2^2(1 -v_2^2) + u_3^2(1-v_3^2) - 2u_2u_3v_2v_3 - 2u_1u_3v_1v_3 - 2u_1u_2v_1v_2 \
                        &= (u_1^2+u_2^2+u_3^2) - (u_1^2v_1^2 + u_2^2v_2^2+u_3^2v_3^2 + 2u_1u_2v_1v_2 + 2u_1u_3v_1v_3 + 2u_2u_3v_2v_3)\
                        &= |u|^2 - langle u,vrangle ^2\
                        &= 1
                        endalign



                        so $u, v, utimes v$ is an orthonormal basis for $mathbbR^3$.






                        share|cite|improve this answer
























                          up vote
                          1
                          down vote













                          Yes, you can check it by direct calculation.



                          Assume that $u = beginbmatrix u_1 \ u_2 \ u_3 endbmatrix$ and $v = beginbmatrix v_1 \ v_2 \ v_3 endbmatrix$ are orthonormal vectors. Their cross product is defined as
                          $$u times v = beginbmatrix u_2v_3 - u_3v_2 \ u_3v_1-u_1v_3 \ u_1v_2 - u_2v_1 endbmatrix$$



                          We have



                          $$langle u, u times vrangle = u_1u_2v_3 - u_1u_3v_2 + u_2u_3v_1 - u_1u_2v_3 + u_1u_3v_2 - u_2u_3v_1 = 0$$



                          $$langle v, u times vrangle = u_2v_1v_3 - u_3v_1v_2 + u_2v_1v_2 - u_1v_2v_3 + u_1v_2v_3 - u_2v_1v_3 = 0$$



                          beginalign
                          |utimes v|^2 &= (u_2v_3 - u_3v_2)^2 + (u_3v_1-u_1v_3)^2 + (u_1v_2 - u_2v_1)^2 \
                          &= u_2^2v_3^2 - 2u_2u_3v_2v_3 + u_3^2v_2^2 + u_3^2v_1^2 - 2u_1u_3v_1v_3 + u_1^2v_3^2 + u_1^2v_2^2 - 2u_1u_2v_1v_2 + u_2^2v_1^2 \
                          &= u_1^2(v_2^2 + v_3^2) + u_2^2(v_1^2 + v_3^2) + u_3^2(v_1^2 + v_2^2) - 2u_2u_3v_2v_3 - 2u_1u_3v_1v_3 - 2u_1u_2v_1v_2 \
                          &= u_1^2(1-v_1^2) + u_2^2(1 -v_2^2) + u_3^2(1-v_3^2) - 2u_2u_3v_2v_3 - 2u_1u_3v_1v_3 - 2u_1u_2v_1v_2 \
                          &= (u_1^2+u_2^2+u_3^2) - (u_1^2v_1^2 + u_2^2v_2^2+u_3^2v_3^2 + 2u_1u_2v_1v_2 + 2u_1u_3v_1v_3 + 2u_2u_3v_2v_3)\
                          &= |u|^2 - langle u,vrangle ^2\
                          &= 1
                          endalign



                          so $u, v, utimes v$ is an orthonormal basis for $mathbbR^3$.






                          share|cite|improve this answer






















                            up vote
                            1
                            down vote










                            up vote
                            1
                            down vote









                            Yes, you can check it by direct calculation.



                            Assume that $u = beginbmatrix u_1 \ u_2 \ u_3 endbmatrix$ and $v = beginbmatrix v_1 \ v_2 \ v_3 endbmatrix$ are orthonormal vectors. Their cross product is defined as
                            $$u times v = beginbmatrix u_2v_3 - u_3v_2 \ u_3v_1-u_1v_3 \ u_1v_2 - u_2v_1 endbmatrix$$



                            We have



                            $$langle u, u times vrangle = u_1u_2v_3 - u_1u_3v_2 + u_2u_3v_1 - u_1u_2v_3 + u_1u_3v_2 - u_2u_3v_1 = 0$$



                            $$langle v, u times vrangle = u_2v_1v_3 - u_3v_1v_2 + u_2v_1v_2 - u_1v_2v_3 + u_1v_2v_3 - u_2v_1v_3 = 0$$



                            beginalign
                            |utimes v|^2 &= (u_2v_3 - u_3v_2)^2 + (u_3v_1-u_1v_3)^2 + (u_1v_2 - u_2v_1)^2 \
                            &= u_2^2v_3^2 - 2u_2u_3v_2v_3 + u_3^2v_2^2 + u_3^2v_1^2 - 2u_1u_3v_1v_3 + u_1^2v_3^2 + u_1^2v_2^2 - 2u_1u_2v_1v_2 + u_2^2v_1^2 \
                            &= u_1^2(v_2^2 + v_3^2) + u_2^2(v_1^2 + v_3^2) + u_3^2(v_1^2 + v_2^2) - 2u_2u_3v_2v_3 - 2u_1u_3v_1v_3 - 2u_1u_2v_1v_2 \
                            &= u_1^2(1-v_1^2) + u_2^2(1 -v_2^2) + u_3^2(1-v_3^2) - 2u_2u_3v_2v_3 - 2u_1u_3v_1v_3 - 2u_1u_2v_1v_2 \
                            &= (u_1^2+u_2^2+u_3^2) - (u_1^2v_1^2 + u_2^2v_2^2+u_3^2v_3^2 + 2u_1u_2v_1v_2 + 2u_1u_3v_1v_3 + 2u_2u_3v_2v_3)\
                            &= |u|^2 - langle u,vrangle ^2\
                            &= 1
                            endalign



                            so $u, v, utimes v$ is an orthonormal basis for $mathbbR^3$.






                            share|cite|improve this answer












                            Yes, you can check it by direct calculation.



                            Assume that $u = beginbmatrix u_1 \ u_2 \ u_3 endbmatrix$ and $v = beginbmatrix v_1 \ v_2 \ v_3 endbmatrix$ are orthonormal vectors. Their cross product is defined as
                            $$u times v = beginbmatrix u_2v_3 - u_3v_2 \ u_3v_1-u_1v_3 \ u_1v_2 - u_2v_1 endbmatrix$$



                            We have



                            $$langle u, u times vrangle = u_1u_2v_3 - u_1u_3v_2 + u_2u_3v_1 - u_1u_2v_3 + u_1u_3v_2 - u_2u_3v_1 = 0$$



                            $$langle v, u times vrangle = u_2v_1v_3 - u_3v_1v_2 + u_2v_1v_2 - u_1v_2v_3 + u_1v_2v_3 - u_2v_1v_3 = 0$$



                            beginalign
                            |utimes v|^2 &= (u_2v_3 - u_3v_2)^2 + (u_3v_1-u_1v_3)^2 + (u_1v_2 - u_2v_1)^2 \
                            &= u_2^2v_3^2 - 2u_2u_3v_2v_3 + u_3^2v_2^2 + u_3^2v_1^2 - 2u_1u_3v_1v_3 + u_1^2v_3^2 + u_1^2v_2^2 - 2u_1u_2v_1v_2 + u_2^2v_1^2 \
                            &= u_1^2(v_2^2 + v_3^2) + u_2^2(v_1^2 + v_3^2) + u_3^2(v_1^2 + v_2^2) - 2u_2u_3v_2v_3 - 2u_1u_3v_1v_3 - 2u_1u_2v_1v_2 \
                            &= u_1^2(1-v_1^2) + u_2^2(1 -v_2^2) + u_3^2(1-v_3^2) - 2u_2u_3v_2v_3 - 2u_1u_3v_1v_3 - 2u_1u_2v_1v_2 \
                            &= (u_1^2+u_2^2+u_3^2) - (u_1^2v_1^2 + u_2^2v_2^2+u_3^2v_3^2 + 2u_1u_2v_1v_2 + 2u_1u_3v_1v_3 + 2u_2u_3v_2v_3)\
                            &= |u|^2 - langle u,vrangle ^2\
                            &= 1
                            endalign



                            so $u, v, utimes v$ is an orthonormal basis for $mathbbR^3$.







                            share|cite|improve this answer












                            share|cite|improve this answer



                            share|cite|improve this answer










                            answered 2 hours ago









                            mechanodroid

                            23.7k52144




                            23.7k52144



























                                 

                                draft saved


                                draft discarded















































                                 


                                draft saved


                                draft discarded














                                StackExchange.ready(
                                function ()
                                StackExchange.openid.initPostLogin('.new-post-login', 'https%3a%2f%2fmath.stackexchange.com%2fquestions%2f2916600%2fforming-an-orthonormal-vector-when-you-already-have-two-perpendicular-vectors%23new-answer', 'question_page');

                                );

                                Post as a guest













































































                                Comments

                                Popular posts from this blog

                                Long meetings (6-7 hours a day): Being “babysat” by supervisor

                                Is the Concept of Multiple Fantasy Races Scientifically Flawed? [closed]

                                Confectionery